GMAT 考满分题库

If Set S = {8, 5, 14, x, 11}, what is the median of Set S?

(1) x > 11

(2) x < 14
  • AStatement (1) ALONE is sufficient, but statement (2) alone is not sufficient.
  • BStatement (2) ALONE is sufficient, but statement (1) alone is not sufficient.
  • CBoth statements TOGETHER are sufficient, but NEITHER one ALONE is sufficient.
  • DEACH statement ALONE is sufficient.
  • EStatements (1) and (2) TOGETHER are NOT sufficient.
显示答案
正确答案: A

讨论题目 或 发起提问

|

题目讨论

  • 按热度
  • 按顺序

最新提问